Đến nội dung

githenhi512 nội dung

Có 276 mục bởi githenhi512 (Tìm giới hạn từ 05-06-2020)



Sắp theo                Sắp xếp  

#615274 $\alpha, \beta$ là hai nghiệm của $x^{2}+2bx+b=1...

Đã gửi bởi githenhi512 on 15-02-2016 - 21:36 trong Đại số

$\Delta =4b^{2}-4b+4> 0\vee b$

$\Rightarrow$ PT luôn có 2 no pb

Theo đ/l Vi-et:   

     $\alpha +\beta =-2b$

     $\alpha .\beta =b-1$

$\Rightarrow \alpha ^{2}+\beta ^{2}=4b^{2}-2b+1$

$\Rightarrow (\alpha -\beta )^{2}=4b^{2}-4b+4=(2b-1)^{2}+3\geq 3$

$\Rightarrow Min(\alpha -\beta )^{2}=3\Leftrightarrow b=1$




#622609 $\frac{1}{(k+1)\sqrt{k}}<2(...

Đã gửi bởi githenhi512 on 25-03-2016 - 21:36 trong Đại số

$\frac{1}{(k+1)\sqrt{k}}=\frac{\sqrt{k}}{k(k+1)}=\sqrt{k}(\frac{1}{k}-\frac{1}{k+1})=\sqrt{k}(\frac{1}{\sqrt{k}}+\frac{1}{\sqrt{k+1}})(\frac{1}{\sqrt{k}}-\frac{1}{\sqrt{k+1}})=(1+\frac{\sqrt{k}}{\sqrt{k+1}})(\frac{1}{\sqrt{k}}-\frac{1}{\sqrt{k+1}})<2(\frac{1}{\sqrt{k}}-\frac{1}{\sqrt{k+1}})$(đpcm)




#639609 $\frac{27a^2}{c(c^2+9a^2)}+\frac{b^2...

Đã gửi bởi githenhi512 on 11-06-2016 - 17:13 trong Bất đẳng thức và cực trị

Bài toán: Cho ba số thực dương $a,b,c$ thỏa mãn $\frac{1}{a}+\frac{2}{b}+\frac{3}{c}=3$. Chứng minh:

$\frac{27a^2}{c(c^2+9a^2)}+\frac{b^2}{a(4a^2+b^2)}+\frac{8c^2}{b(9b^2+4c^2)}\geq \frac{3}{2}$

$\frac{27a^2}{c(c^2+9a^2)}=\frac{3}{c}.(1-\frac{c^2}{c^2+9a^2})\geq \frac{3}{c}(1-\frac{c^2}{6ac})=\frac{3}{c}-\frac{1}{2a}$

$\frac{b^2}{a(4a^2+b^2)}=\frac{1}{a}(1-\frac{4a^2}{4a^2+b^2})\geq \frac{1}{a}(1-\frac{4a^2}{4ab})=\frac{1}{a}-\frac{1}{b}$

$\frac{8c^2}{b(9b^2+4c^2)}=\frac{2}{b}.(1-\frac{9b^2}{9b^2+4c^2})\geq \frac{2}{b}(1-\frac{9b^2}{12bc})=\frac{2}{b}-\frac{3}{2c}$

Do đó: $VT\geq \frac{1}{2a}+b+\frac{3}{2c}=VP(đpcm)$




#628832 $\frac{a-bc}{a+bc}+\frac{b-ca}...

Đã gửi bởi githenhi512 on 21-04-2016 - 21:39 trong Bất đẳng thức và cực trị

$B=\frac{3-A}{2}=\sum \frac{bc}{a+bc}$=$\sum \frac{bc}{(a+b)(a+c)}$

Cần CM A$\leq$ 1.5 $\Leftrightarrow B\geq \frac{3}{4}$

$\Leftrightarrow \sum \frac{bc}{(a+b)(a+c)}\geq \frac{3}{4}$

$\Leftrightarrow 4bc(b+c)+4ca(c+a)+4ab(a+b)\geq 3(a+b)(b+c)(c+a)$

$\Leftrightarrow a^{2}b+ab^{2}+b^{2}c+bc^{2}+c^{2}a+ca^{2}\geq 6abc$(luôn đ)

$\Rightarrow$ đpcm




#627888 $\frac{ab}{4b+4c+a}+\frac{bc}...

Đã gửi bởi githenhi512 on 17-04-2016 - 23:40 trong Bất đẳng thức và cực trị

http://diendantoanho...bcleq-fracabc9/




#630901 $\frac{x^{4}+1}{x^{3}-x}...

Đã gửi bởi githenhi512 on 02-05-2016 - 21:59 trong Bất đẳng thức và cực trị

Cho x thuộc R thỏa mãn x > 1. CM

$\frac{x^{4}+1}{x^{3}-x}\geq 2\sqrt{2}$

x>1 $\Rightarrow x^{3}-x>0$

Cần CM: $x^{4}-2\sqrt{2}x^{3}+2\sqrt{2}x+1\geq 0\Leftrightarrow (x^{2}-1-\sqrt{2}x)^{2}\geq 0$(đ)

Dấu ''='' xr $\Leftrightarrow x=\frac{\sqrt{2}+\sqrt{6}}{2}$




#638800 $\left ( x^{2} +3x+5\right )\sqrt{8x^...

Đã gửi bởi githenhi512 on 07-06-2016 - 21:20 trong Phương trình - hệ phương trình - bất phương trình

Đề đúng rồi bạn,....nếu 21 bạn dùng pp gì vậy?

$\Leftrightarrow \sqrt{8x^2+4x+15}-(x+3)=\frac{x^3+13x^2+12x+21}{x^2+3x+5}-(x+3)\Leftrightarrow \frac{7x^2-2x+6}{\sqrt{8x^2+4x+15}+x+3}=\frac{7x^2-2x+6}{x^2+3x+5}$

$Mà 7x^2-2x+6>0\Rightarrow \sqrt{8x^2+4x+15}+x+3=x^2+3x+5$ 

Đến đây thì tắc rồi :(




#638390 $\left ( x^{2} +3x+5\right )\sqrt{8x^...

Đã gửi bởi githenhi512 on 05-06-2016 - 21:41 trong Phương trình - hệ phương trình - bất phương trình

$\left ( x^{2} +3x+5\right )\sqrt{8x^{2}+4x+15}=x^{3}+13x^{2}+12x+31$

Chỗ này là 31 hay 21 vậy? 21 có vẻ đúng hơn :icon6:




#700089 $\left\{\begin{matrix} u(1)=\sqrt...

Đã gửi bởi githenhi512 on 11-01-2018 - 17:16 trong Dãy số - Giới hạn

 

Số hạng tổng quát $u(n)=2Cos\frac{\pi }{2^{n+1}}$ hoàn toàn có thể chứng minh bằng quy nạp. Nhưng mình thắc mắc là tại sao lại tìm được công thức đấy?

Cho dãy số xác định: $\left\{\begin{matrix} u(1)=\sqrt{2} & & \\ u(n+1)=\sqrt{2+u(n)} & & \end{matrix}\right.$
Tìm số hạng tổng quát của dãy số.




#623301 $\left\{\begin{matrix} x^{2}-2xy...

Đã gửi bởi githenhi512 on 28-03-2016 - 21:59 trong Phương trình, hệ phương trình và bất phương trình

:icon6:

File gửi kèm




#628855 $\left\{\begin{matrix} x^{3}+4y=...

Đã gửi bởi githenhi512 on 21-04-2016 - 22:09 trong Phương trình, hệ phương trình và bất phương trình

$21x^{2}-5xy-4y^{2}=0$

Cái $(x^{2}-5xy+16)=0$  giải quyết thế nào

Thay 4=y2-5x2  tđ: $21x^{2}-5xy-4y^{2}=0$

N $x\neq 0\Rightarrow 21-5\frac{y}{x}-4(\frac{y}{x})^{2}=0\Rightarrow$




#632185 $\left\{\begin{matrix}\sqrt{x+2...

Đã gửi bởi githenhi512 on 09-05-2016 - 23:34 trong Phương trình, hệ phương trình và bất phương trình

b) Giải phương trình sau : $\sqrt{3y^2-6y-6} = 3\sqrt{(2-x)^5} + (7x-19)\sqrt{2-x}$

Chỗ đó phải là $3x^2-6x-6$ chứ!

Đk: $x\leq 1-\sqrt{3}$

$\Leftrightarrow \sqrt{3x^2-6x-6}=(3x^2-5x-7)\sqrt{2-x}$

Đ $\sqrt{3x^2-6x-6}=a\geq 0, \sqrt{2-x}=b>0\Rightarrow a=(a^2-b^2+1)b\Leftrightarrow (a-b)(1-ab-b^2)=0$

N a=b thì x=-1

N $1-ab-b^2=0\Rightarrow 0\leq ab=1-b^2=x-1>0(l)$




#631100 $\left\{\begin{matrix}(x-y-1)\sqrt...

Đã gửi bởi githenhi512 on 03-05-2016 - 22:55 trong Phương trình - hệ phương trình - bất phương trình

1. $\left\{\begin{matrix}x^2+(y^2-y-1)\sqrt{x^2+2}-y^3+y+2=0\ (1) \\ \sqrt[3]{y^2-3}-\sqrt{xy^2-2x-2}+x=0\ (2)\end{matrix}\right.$

$(1)\Leftrightarrow y^2=x^2+2$
Khúc sau pt(2) thế vào thì mình làm không được mong mọi người chỉ

 

$\Leftrightarrow \sqrt[3]{x^{2}-1}-\sqrt{x^{3}-2}+x=0(x\geq \sqrt[3]{2})$

$\Leftrightarrow \sqrt[3]{x^{2}-1}-(x-1)+(2x-1)-\sqrt{x^{3}-2}=0\Leftrightarrow \frac{\sqrt[3]{x-1}.x.(3-x)}{\sqrt[3]{(x+1)^{2}}+\sqrt[3]{(x+1)(x-1)^{2}}+\sqrt[3]{(x-1)^{4}}}+\frac{(3-x)(x^{2}-x+1)}{2x+1+\sqrt{x^{3}+2}}=0$

$x\geq \sqrt[3]{2}>1\Rightarrow x=3\Rightarrow ..$




#623886 $\left\{\begin{matrix}x+y\sqrt{x...

Đã gửi bởi githenhi512 on 31-03-2016 - 20:42 trong Phương trình, hệ phương trình và bất phương trình

b. x=y2 $\Rightarrow x\geq 0$

(2) $\Leftrightarrow (\sqrt{4x+5}-3)+(\sqrt{x^{2}+8}-3)=0$

$\Leftrightarrow \frac{4(x-1)}{\sqrt{4x+5}+3}+\frac{(x-1)(x+1)}{\sqrt{x^{2}+8}+3}=0$

x=1 là nghiệm của pt. N x$\neq 0$ 

$\Rightarrow \frac{4}{\sqrt{4x+5}+3}+\frac{x+1}{\sqrt{x^{2}+8}+3}=0$

VT$\geq 0\vee x\geq 0$ nên x=1 là nghiệm duy nhất $\Rightarrow y=-1;1$




#623941 $\left\{\begin{matrix} xy &-\frac{x}{y}=\fr...

Đã gửi bởi githenhi512 on 31-03-2016 - 22:36 trong Phương trình, hệ phương trình và bất phương trình

Đk: x,y$\neq$0.

$\frac{y}{x}-\frac{x}{y}=\frac{5}{6}$

$\Leftrightarrow 6y^{2}-5xy-6x^{2}=0$

$x\neq 0\Rightarrow 6(\frac{y}{x})^{2}-5\frac{y}{x}-6=0$

N $\frac{y}{x}=-1\Rightarrow ...$

N $\frac{y}{x}=\frac{3}{2}\Rightarrow ...$




#685519 $\sqrt{3x^3+2x^2+2}+\sqrt{-3x^3+x^2+2x-1}=...

Đã gửi bởi githenhi512 on 25-06-2017 - 01:11 trong Phương trình, hệ phương trình và bất phương trình

Giải phương trình

$\sqrt{3x^3+2x^2+2}+\sqrt{3x^3+x^2+2x-1}=2x^2+2x+2$

Cách 1:

 Đk: $3x^3+2x^2+2, -3x^3+x^2+2x-1\geq 0\Rightarrow x< -0.5$

Ta có: $2x^2+2x+2=\sqrt{3x^3+2x^2+2}+\sqrt{3x^3+x^2+2x-1}\leq \sqrt{2(3x^2+2x+1)}\Leftrightarrow 2x^2+4x+2\leq \sqrt{2(3x^2+2x+1)}+2x\Leftrightarrow 2x^2+4x+2\leq \frac{2x^2+4x+2}{\sqrt{2(3x^2+2x+1)}-2x}$

Mặt khác: $\sqrt{2(3x^2+2x+1)}-2x> 1$( do $x< -0.5$) $\Leftrightarrow 2x^2+4x+2\geq \frac{2x^2+4x+2}{\sqrt{2(3x^2+2x+1)}-2x}$

Do đó: $2x^2+4x+2=0\Leftrightarrow x=-1$

Cách 2: 

Từ Đk $\rightarrow x\in (-1.16;-0.8)$

pt $\Leftrightarrow [\sqrt{3x^3+2x^2+2}-(2.5x+3.5)]+[\sqrt{-3x^3+x^2+2x-1}-(-4.5x-3.5)]=2x^2+4x+2$

$\Leftrightarrow \frac{(x+1)^2(3x-10.25)}{\sqrt{3x^3+2x^2+2}+(2.5x+3.5)}+\frac{-(x+1)^2(3x+13.25)}{\sqrt{-3x^3+x^2+2x-1}+(-4.5x-3.5)}=(x+1)^2$

Nhận thấy: $VT\leq 0, VP\geq 0\veebar x\in (-1,16;-0.8)\Rightarrow x=-1$




#632346 $\sqrt{5x-1}-\sqrt{3x+13}=\frac{...

Đã gửi bởi githenhi512 on 10-05-2016 - 21:23 trong Phương trình, hệ phương trình và bất phương trình

Giải pt $\sqrt{5x-1}-\sqrt{3x+13}=\frac{x-7}{3}$

Đk: $x\geq 0.2$

$\frac{x-7}{3}=\sqrt{5x-1}-\sqrt{3x+13}=\frac{2(x-7)}{\sqrt{5x-1}+\sqrt{3x+13}}$

N x=7(t/m)

N $\sqrt{3x+13}+\sqrt{5x-1}=6\Rightarrow \sqrt{5x-1}=6-\sqrt{3x+13}\Rightarrow ..$




#682857 $\sqrt{x+1}+\sqrt{y+1}+\sqrt{z+1...

Đã gửi bởi githenhi512 on 03-06-2017 - 10:34 trong Bất đẳng thức - Cực trị

Cho x, y, z $\in [-1;1]$ thỏa mãn: x+ y+ z+ xyz= 0. Chứng minh: $\sqrt{x+1}+\sqrt{y+1}+\sqrt{z+1}\leq 3$




#624327 $\sqrt{x^{2}-2m}+2\sqrt{x^{2...

Đã gửi bởi githenhi512 on 02-04-2016 - 20:47 trong Phương trình, hệ phương trình và bất phương trình

Câu a đề là sao z?




#628869 $\sqrt[3]{abc}+\sqrt[3]{xyz}\leq...

Đã gửi bởi githenhi512 on 21-04-2016 - 23:29 trong Bất đẳng thức và cực trị

bạn có thể chứng minh BĐT bạn dung ko?

Bđt Cauchy: $\sqrt[3]{abc}\leq a+b+c$(a,b,c>0)

Dấu ''='' xr khi a=b=c>0




#628862 $\sqrt[3]{abc}+\sqrt[3]{xyz}\leq...

Đã gửi bởi githenhi512 on 21-04-2016 - 22:53 trong Bất đẳng thức và cực trị

a)Cho x,y,z,a,b,c>0 .CMR:

$\sqrt[3]{abc}+\sqrt[3]{xyz}\leq \sqrt[3]{(a+x)(b+y)(c+z)}$

b) từ a) suy ra:

$\sqrt[3]{3+\sqrt[3]{3}}+\sqrt[3]{3-\sqrt[3]{3}}\leq 2\sqrt[3]{3}$

a. $\frac{\sqrt[3]{abc}+\sqrt[3]{xyz}}{\sqrt[3]{(a+x)(b+y)(c+z)}}$

$=\sqrt[3]{\frac{a}{a+x}.\frac{b}{b+y}.\frac{c}{c+z}}+\sqrt[3]{\frac{x}{a+x}.\frac{y}{b+y}.\frac{z}{c+z}}.$

$\leq \frac{1}{3}(\frac{a}{a+x}+\frac{b}{b+y}+\frac{c}{c+z}+\frac{x}{a+x}+\frac{y}{b+y}+\frac{z}{z+c})=1$

(đpcm)

b. $VT=\sqrt[3]{\sqrt[3]{3}.(\sqrt[3]{3^{2}}+1).1}+\sqrt[3]{\sqrt[3]{3}.(\sqrt[3]{3^{2}}-1).1}.$

            $\leq \sqrt[3]{2.\sqrt[3]{3}.2\sqrt[3]{3^{2}}.2}=VP.$(đpcm)




#647783 $\sqrt[4]{1-x^2}+\sqrt[4]{1+x}+\sqrt[4]{1-x}=3$

Đã gửi bởi githenhi512 on 03-08-2016 - 16:27 trong Phương trình, hệ phương trình và bất phương trình

1)

ĐK: x$\geq 0$

đặt $t=\sqrt[4]{x} (t\geq 0)$

pt đã cho tương đương với

$2t^{4}-t+\frac{3}{8}=0$

$\Leftrightarrow (t-\frac{1}{2})(2t^{3}+t^{2}+0,5t-\frac{3}{4})=0$

$\Leftrightarrow t=0,5$ (chọn)

$\Leftrightarrow x=\sqrt[4]{\frac{1}{2}}$ (chọn) 

Còn 1 no $t=\frac{1}{2}\Leftrightarrow x=\frac{1}{16}$




#627881 $\sum (\frac{a^4}{(a+2)(b+2)})>=\frac{1}{3}$

Đã gửi bởi githenhi512 on 17-04-2016 - 23:25 trong Bất đẳng thức và cực trị

chứng minh rằng \sum (\frac{a^4}{(a+2)(b+2)})>=\frac{1}{3}

$\sum a^{2}\geq \frac{(\sum a)^{2}}{3}=3$

$VT\geq \frac{(\sum a^{2})^{2}}{\sum (a+2)(b+2)}$

       =$\frac{(\sum a^{2})^{2}}{\sum ab+4\sum a+12}$

       $\geq \frac{(\sum a^{2})^{2}}{\sum a^{2}+24}$

Cần CM: $\frac{(\sum a^{2})^{2}}{24+\sum a^{2}}\geq \frac{1}{3}$

$\Leftrightarrow (3\sum a^{2}+8)(\sum a^{2}-3)\geq 0$(luôn đ)(đpcm)

Dấu ''='' xr khi a=b=c=1




#685516 $(x^3-4)=(\sqrt[3]{(x^2+4)^2}+4)^2$

Đã gửi bởi githenhi512 on 24-06-2017 - 23:36 trong Phương trình - hệ phương trình - bất phương trình

Bài toán: Giải phương trình:

$$(x^3-4)^3=(\sqrt[3]{(x^2+4)^2}+4)^2$$

Từ pt $\Rightarrow x^3>4$

pt $\Leftrightarrow (x^3-4)^3-x^6=[\sqrt[3]{(x^2+4)^2}+4]^2-x^6\Leftrightarrow (x-2)(x^2+x+2)[(x^3-4)^2+(x^3-4)x^2+x^4]=[\sqrt[3]{(x^2+4)^2}+4-x^3][\sqrt[3]{(x^2+4)^2}+4+x^3]$

Mà $\sqrt[3]{(x^2+4)^2}+4-x^3=\frac{(2-x)[x^4(x-2)(x^3+2x^2+4x+4)+x^3(4x^3-15)+4x^6+18x^2+20x+40]}{\sqrt[3]{(x^2+4)^4}+\sqrt[3]{(x^2+4)^2}(x^3-4)+(x^3-4)^2}\leq 0\veebar x^3>4$

Do đó: $x= 2.$




#618698 ..

Đã gửi bởi githenhi512 on 06-03-2016 - 12:18 trong Hình học

Cho tam giác ABC cân tại A, đường tròn (O) tiếp xúc với AB và AC tại B và C. Lấy M thuộc cung BC nhỏ. Kẻ MD vuông góc với BC, MẸ  vuông góc với AC,MF vuông góc với AB. Xác định vị trí điểm M để MD.ME.MF đạt giá trị lớn nhất.